Another convergence/divergence question

  • Thread starter philosophking
  • Start date
In summary: I'm not sure what you mean when you say "a theorem states that if a series converges, then the limit is zero." Maybe I can clarify things for you if you can post that theorem.-joebooIn summary, the conversation discusses a practice problem for a mathematics competition and whether or not a particular series converges or diverges. The participants present different approaches and arguments, including the use of logarithms and inequalities. In the end, it is concluded that the series does converge, with one participant offering a detailed explanation and clarification of the concept of sequences versus series.
  • #1
philosophking
175
0
Another convergence/divergence question :)

Hi, thank you in advance for your help. This was a practice problem for the New Jersey Undergraduate Mathematics Competition last year.

Prove whether or not the following converges:

[tex]\sqrt{1+\sqrt{2+\sqrt{3+...}}}[/tex]

Forgive my LaTeX, I'm still learning :). Suggestions on that would be appreciated as well, haha.

For this problem, I was thinking of reducing it to the following:

[tex]\sqrt{\sqrt\ ... \sqrt{n}}[/tex]

which reduces to [tex]n^{1/n}[/tex],

which I would then show diverges, because I think it does diverge, but I'm just not sure.

Thanks for the help.
 
Last edited:
Physics news on Phys.org
  • #2
Ok, here's what I think ( if you think you're getting the idea halfway thru, stop reading as I post a full solution ):

[tex]A, B \geq 1 \longrightarrow 0 < \frac{1}{A} + \frac{1}{B} \leq 2[/tex]

This gives:
[tex]2 \leq A+B \leq 2A \cdot B[/tex]

[itex]\ln(x)[/itex] increasing on [itex](0, +\infty )[/itex] gives:

[tex]\ln(2) \leq \ln(A+B) \leq \ln (2A \cdot B) = \ln(2) + \ln(A) + \ln(B)[/tex]

Now define:
[tex]S(n) = \sqrt{1+\sqrt{2+\sqrt{3 + \dots +\sqrt{n}}}}[/tex]

Now, I claim, by taking the logarithm of the [itex]S(n)[/itex], and repeatedly applying the above inequality, we obtain:
[tex] 0 \leq \ln(S(n)) \leq -\frac{\ln(2)}{2^n} + \sum_{k=1}^n \frac{\ln(2) + \ln(k)}{2^k}[/tex]
or:
[tex] 1 \leq S(n) \leq e^{-\frac{\ln(2)}{2^n} + \sum_{k=1}^n \frac{\ln(2) + \ln(k)}{2^k}} [/tex]

Letting [itex] n \rightarrow \infty [/itex] gives the desired result ( noting that the series in the exponent will converge. )

( hope that's right, hehe )

-joeboo

edit: I just wanted to add 1 thing that may prevent some small confusion-
the [itex]-\frac{\ln(2)}{2^n}[/itex] is caused because the last term in the series doesn't get a "ln(2)" term added in because we are not applying the inequality ( ie, [itex]\ln(\sqrt{4})[/itex] in the S(4) expansion doesn't need to be dealt with, so there will be no accompanying ln(2) term ) - I hope that helps
 
Last edited:
  • #3
I'm sorry, this seems very complicated. Is there anyway you could elucidate?

What do you think of my idea? Of taking some slightly smaller sum ([tex]n^{1/n}[/tex]), trying to show that the limit of that does not approach zero, and concluding that the bigger sum does not converge either? It seems pretty easy to take the limit of what that is, but I just don't know how to do it.

Thanks again for your help.
 
  • #4
What I'm saying in my previous post is that it converges.
 
  • #5
I know you are.

I was just wondering if you could either elucidate your argument or criticize my argument?

Thank you again for the help.
 
  • #6
Sorry.

I'll try and elaborate.
You're trying to determine if:
[tex]S = \sqrt{1+\sqrt{2+\sqrt{3+\dots }}}[/tex]
Exists or not.

All I'm doing is establishing a new sequence,

[tex]S(n)=\sqrt{1+\sqrt{2+\sqrt{3+\dots +\sqrt{n}}}}[/tex]

such that:
[tex]\lim_{n \rightarrow \infty} S(n) = S[/tex]

Clearly, [itex]S(n)[/itex] is increasing. Then I show that it is bounded below by [itex]1[/itex] and bounded above by:

[tex]e^{-\frac{\ln(2)}{2^n} + \sum_{k=1}^n \frac{\ln(2) + \ln(k)}{2^k}}[/tex]

I arrive at that conclusion by applying the inequalities with the logarithm over and over to the [itex]S(n)[/itex] terms
Now, the exponent is composed of 2 terms:

[tex]-\frac{\ln(2)}{2^n}}[/tex]

and:
[tex]\sum_{k=1}^n \frac{\ln(2) + \ln(k)}{2^k}[/tex]

the first term goes to [itex]0[/itex] as [itex]n \rightarrow \infty[/itex] and the second series converges as well ( root test, ratio test both work well ).
Therefore, the sequence [itex]S(n)[/itex] is convergent as it is a monotonic sequence bounded by 2 convergent sequences.

As far as critiquing your argument, I'm not entirely sure I understand it.
philosophking said:
Of taking some slightly smaller sum ([itex]n^\frac{1}{n}[/itex]), trying to show that the limit of that does not approach zero, and concluding that the bigger sum does not converge either
I'm confused here because in the original statement of the problem, there is no infinite sum. It's a "continued radical" ( if there is such a term ).
If you are suggesting the use of:
[tex]\sqrt{n^\frac{1}{n} +\sqrt{n^\frac{1}{n} +\sqrt{n^\frac{1}{n} +\dots}}}[/tex]
in some fashion I would suggest you consider the following:
[tex]\sqrt{1+\sqrt{1+\sqrt{1+\dots}}} = \frac{1+\sqrt{5}}{2}[/tex]

Sorry if I'm making this seem too complicated. If you want to be more specific with your method ( as I'm not sure what you're suggesting ), I'd happily go into more detail.

-joeboo
 
  • #7
Well [tex] \lim_{x\rightarrow \infty} n^{1/n} [/tex] does equal 1, if that helps at all. This would imply divergence, assuming the original assumption is correct, which I'm not sure it is.

Let [tex]y=n^{1/n}[/tex]
[tex]\ln y=\frac{1}{n} \ln n=\frac{\ln n}{n}[/tex]
[tex]\lim_{x\rightarrow \infty} \ln y=\lim_{x\rightarrow \infty} \frac{\ln n}{n}=\lim_{x\rightarrow \infty}\frac{1}{n}=0[/tex]
[tex]\lim_{x\rightarrow \infty} y=e^0=1[/tex]
 
  • #8
Yeah, a quick check with a calculator will in fact show that this will converge to about 1.758ish.
 
  • #9
Thank you joe, I apologize for having you type most of your stuff out again, but that definitely helps, and I see where you obtained your expressions. I see how at the end you're showing that since the original expression is between two things that converge, it also converges, and that's how you come to your conclusion.

I guess I just have to figure out where I went wrong. This is how I see the problem:

[tex]\sqrt{1+\sqrt{2+\sqrt{3+... \sqrt{n}}}}\geq \sqrt{\sqrt{\sqrt{...\sqrt{n}}}}[/tex]

And the RHS is basically n square roots, i.e., [tex]n^{1/n}[/tex] is what the RHS is. Now, a theorem states that if a series converges, then the limit is zero. I.e., if the limit is not zero, the series does not converge (diverges). Also, if one series is bigger than the other, and the smaller diverges, we know the larger one diverges as well.

Hence, if we can show that limit as n->infinity of the RHS is not zero, we could show that the series diverges, which implies the original diverges.

Where do I go wrong? Is it in my assumption that it is an "infinite sum" or whatever, as you said?
 
  • #10
does it make sense to say that since

[tex] \lim_{x \rightarrow \infty} 1 = 1 \neq 0[/tex]

that the sequence [itex]\{1\}[/itex] diverges?

Indeed, the theorem you refer to deals with series, not sequences.

If we let

[tex]x_n=\sqrt{1+\sqrt{2+\sqrt{3+... \sqrt{n}}}}[/tex]

then you are trying to find

[tex]\lim_{n\rightarrow \infty} x_n[/tex]

ie. the limit of a sequence, not a series.

Your inequality just tells you that if the sequence converges, then it converges to something [itex]\geq 1[/itex]
 
  • #11
philosophking said:
Hi, thank you in advance for your help. This was a practice problem for the New Jersey Undergraduate Mathematics Competition last year.

Prove whether or not the following converges:

[tex]\sqrt{1+\sqrt{2+\sqrt{3+...}}}[/tex]

Forgive my LaTeX, I'm still learning :). Suggestions on that would be appreciated as well, haha.

For this problem, I was thinking of reducing it to the following:

[tex]\sqrt{\sqrt\ ... \sqrt{n}}[/tex]

which reduces to [tex]n^{1/n}[/tex],

which I would then show diverges, because I think it does diverge, but I'm just not sure.

Thanks for the help.

[tex]S1(4) = \sqrt{1+\sqrt{2+\sqrt{3+\sqrt{4}}}}[/tex]
[tex]S2(4) = \sqrt{1+\sqrt{2+\sqrt{3+4}}}[/tex]

[tex]\sqrt{1+\sqrt{2+\sqrt{3+\sqrt{4}}}} <= \sqrt{1+\sqrt{2+\sqrt{3+4}}}[/tex]
[tex]S1 <= S2[/tex]

[tex]\sqrt{1+\sqrt{2+\sqrt{3+4}}} < \sqrt{1+\sqrt{2+3}}[/tex]
This behavior continues as n increases, with each term being slightly smaller. This is because x < sqr(2x + 1) when:
x=sqr(2x+1)
x^2 = 2x + 1
x^2 - 2x - 1 = 0
x = 2.4142
So it must get smaller when n increases once n is greater than or equal to 3.
[tex]S2(n) > S2(n+1)[/tex]

[tex]\sqrt{1+\sqrt{2+\sqrt{3+\sqrt{4}}}} >= \sqrt{1+\sqrt{2+\sqrt{3}}}[/tex]
This is obviously true, since we're adding more. Each term will be slightly larger.
[tex]S1(n) < S1(n+1)[/tex]

So we have
[tex]S1(n+1) > S1(n)[/tex]
[tex]S2(n+1) < S2(n)[/tex]
[tex]S1 <= S2[/tex]

S1 can't go past S2 and it must increase. S2 can't go past S1 and it must decrease. Looks like convergence to me!
 
Last edited:
  • #12
philosophking said:
[tex]\sqrt{1+\sqrt{2+\sqrt{3+... \sqrt{n}}}}\geq \sqrt{\sqrt{\sqrt{...\sqrt{n}}}}[/tex]

And the RHS is basically n square roots, i.e., [tex]n^{1/n}[/tex] is what the RHS is.

The RHS is not [tex]n^{1/n}[/tex], it's [tex](((n^{1/2})^{1/2})\ldots)^{1/2}=n^{1/2^n}[/tex].

In anycase [tex]n^{1/n}[/tex] goes to 1 as n goes to infinity, so the best you could say is if your orignial series converges, then it converges to something greater than or equal to 1.

You can also bound your original sequence by 2 without too much trouble. Take [tex]x_{1}=2[/tex] and define [tex]x_{i+1}=x_{i}^2-i[/tex] for all i>0. (note that we have [tex]x_{i}=\sqrt{i+x_{i+1}}[/tex] as well). Show by induction that [tex]x_{i}^2> 2i[/tex] for all i. Now to show S(n) (using the same notation as joeboo) is less than 2 consider [tex]x_{n}[/tex] and work backwards:

[tex]\sqrt{n}\leq x_{n}[/tex]

[tex]\sqrt{(n-1)+\sqrt{n}}\leq\sqrt{(n-1)+x_{n}}=x_{n-1}[/tex]

... (use a finite induction here)

[tex]S(n)=\sqrt{1+\sqrt{2+\sqrt{3+\dots +\sqrt{n}}}}\leq x_{1}=2[/tex]
 
  • #13
It's convergence is pretty simple to show N^(1/N) = (1)/(N^N) if you take the limit as N goes to infinity it is 0 and if the limit of a sequence as N goes to infinity is some number the sequence converges.
 
  • #14
[tex]n^{\frac{1}{n}} \neq n^{-n}[/tex]

...
 
  • #15
Alright, I'm pretty convinced. I wrote a Mathematica program to plot the trend up to n=25 (you know, nested 25 times and all the ones before it). It seems to jump pretty quick to a value around 1.75793. See the attached plot. Can anyone prove this?

Ok, I see T!M already showed this. A plot is Ok too though.
 

Attachments

  • nestedroot.JPG
    nestedroot.JPG
    4.1 KB · Views: 357
Last edited:
  • #16
According to Herschfield's Convergence Theorem (which I just learned):

[tex]\sqrt{x_1+\sqrt{x_2+\sqrt{x_3+...}}}[/tex]

converges iff the sequence:

[tex]\{(x_n)^{1/{2^n}}\}[/tex]

is bounded.

So, for positive terms, it's bounded below by 0 and for:

[tex]\sqrt{1+\sqrt{2+\sqrt{3+...}}}[/tex]

we have:

[tex](x_n)^{1/{2^n}}=n^{1/{2^n}}<(2^n)^{1/{2^n}}=2^{n/{2

^n}}[/tex]

and:

[tex]\lim_{n\rightarrow\infty}2^{n/{2^n}}=2^0=1[/tex]

Thus the sequence is bounded and therefore the nested square root converges.
 
  • #17
Very interesting! Nice insight. What class did you learn that in, analysis?

Thanks for your help.
 
  • #18
philosophking said:
Very interesting! Nice insight. What class did you learn that in, analysis?

Thanks for your help.

Well, I just reviewed Herschfield's theorem this morning via MathWorld and then applied it to the problem. I believe my analysis is correct but leave it up to you and others reviewing my post to verify or point out flaws.

Also, reviewing the web for nested radicals lead me to believe that determining the value of the limit analytically is not easily done and in fact may not be possible although I yield to anyone who proves me wrong.
 

1. What is convergence and divergence in science?

Convergence and divergence refer to the tendency of different species to become more or less similar over time. Convergence occurs when unrelated species evolve similar traits to adapt to similar environmental conditions, while divergence occurs when closely related species evolve different traits due to different environmental pressures.

2. What are some examples of convergence and divergence in nature?

Examples of convergence include the development of wings in birds, bats, and insects for flight, and the evolution of similar body shapes in dolphins and sharks for efficient swimming. Divergence can be seen in the variations in beak shape and size among finches on the Galapagos Islands, as well as the differences in camouflage patterns among different species of chameleons.

3. How do scientists study convergence and divergence?

Scientists study convergence and divergence by comparing the physical and genetic characteristics of different species. This can involve analyzing DNA sequences, studying fossil records, and observing the behavior and adaptations of living organisms in their natural habitats.

4. What is the significance of convergence and divergence in evolutionary biology?

Convergence and divergence are important concepts in evolutionary biology as they help us to understand how species adapt and evolve in response to their environment. It also provides evidence for common ancestry and the process of natural selection, as similar traits can arise independently in different species due to similar environmental pressures.

5. Can convergence and divergence occur in non-biological systems?

Yes, convergence and divergence can also occur in non-biological systems, such as in technology and culture. For example, different cultures may develop similar technological advancements due to similar needs, leading to convergence. On the other hand, factors such as geographical isolation can result in cultural divergence, leading to the development of unique traditions and practices.

Similar threads

Replies
15
Views
2K
Replies
11
Views
2K
Replies
3
Views
936
  • Calculus
Replies
2
Views
1K
Replies
6
Views
2K
  • Calculus
Replies
1
Views
1K
  • Calculus
Replies
1
Views
997
Replies
18
Views
2K
Replies
4
Views
1K
Replies
14
Views
2K
Back
Top